LSAT and Law School Admissions Forum

Get expert LSAT preparation and law school admissions advice from PowerScore Test Preparation.

 sunj289
  • Posts: 7
  • Joined: May 14, 2013
|
#9492
Hi,
LR section 1, Q 17:

I'm having a difficult time understanding why D is necessarily the correct answer. Perhaps i'm not understanding the stimulus correctly, but I'm just not able to fully grasp this problem.

Thanks in advance!
 Ron Gore
PowerScore Staff
  • PowerScore Staff
  • Posts: 220
  • Joined: May 15, 2013
|
#9503
Thank you for your question, sunj289!

This is a Resolve the Paradox question. To attack this question, you first identify the contradiction. Then you look for an answer choice that could permit or cause both seemingly contradictory facts to be true.

Here, the contradictory situation is that even though a large amount of economic activity occurs in these large malls, the increase in the local economy is smaller than the total amount of economic activity in the malls. This situation runs counter to the expectation described in the first clause of the first sentence, that you would expect all of that economic activity in the mall to boost the local economy.

This stated expectation is based on an assumption, that the presence of the mall creates new economic activity in the area, and doesn't simply siphon off economic activity from the local economy that would have occurred even in the absence of the mall. Notice that the stimulus refers to "economic activity," a broad idea, and not simply dollars spent by consumers; the incorrect answers will play around with this broad concept of economic activity.

Answer choice (D) helps to resolve the apparent discrepancy by revealing that most of the money spent in the mall is money that would have been spent in the same community even in the absence of the mall. So, while the economic activity is new to the mall, it is not new to the community. That is why the increase in the local economy is much smaller. The further implication of this answer is that other businesses in the community are worse off, because the money that used to be spent in their stores is now being spent in the mall.

In looking at the other answer choices, remember that an incorrect answer in a resolve the paradox question will: explain only one side of the paradox, misstate the facts, or try to explain some other aspect of how the facts relate to each other that is immaterial to the discrepancy.

Answer (A) misstates the facts, because it implies there is not a large amount of economic activity at the mall.

Answer (B) explains the expectation that the opening of the mall would boost the local economy, but doesn't explain the contradiction between that expectation and reality.

Answer (C), like (B), explains the expectation that the opening of the mall would boost the local economy, but doesn't explain the contradiction.

Answer (E), like (A), attacks the facts, by implying there is not a large amount of economic activity, i.e., new well-paying jobs, at the mall.

Hope that helps,

Ron
 srcline@noctrl.edu
  • Posts: 243
  • Joined: Oct 16, 2015
|
#23700
Hello Ron,

Thankyou for your explanation, especially with the discrepancy and the incorrect answer choices. However, I am still having a hard time understanding why B is not correct. In your explanation you stated that it only explains the half of the discrepancy and that it doesn't explain the contradiction b/w the expectation and reality.

So my thought processes on B was that: Given that tourists are visiting these malls and spending most of their money at these malls, wouldn't that explain why the total amount of the econ. activity is higher in these malls. And also explain why the increase in the local econ is smaller b/c tourists are not visiting other attractions in the local community like other businesses, b/c they came specifically for that mall.

Is the discrepancy getting at a long term impact? If so I don't see where that is hinted in the stimulus . I thought D was too ambiguous.

Thankyou
Sarah
 David Boyle
PowerScore Staff
  • PowerScore Staff
  • Posts: 836
  • Joined: Jun 07, 2013
|
#23789
srcline@noctrl.edu wrote:Hello Ron,

Thankyou for your explanation, especially with the discrepancy and the incorrect answer choices. However, I am still having a hard time understanding why B is not correct. In your explanation you stated that it only explains the half of the discrepancy and that it doesn't explain the contradiction b/w the expectation and reality.

So my thought processes on B was that: Given that tourists are visiting these malls and spending most of their money at these malls, wouldn't that explain why the total amount of the econ. activity is higher in these malls. And also explain why the increase in the local econ is smaller b/c tourists are not visiting other attractions in the local community like other businesses, b/c they came specifically for that mall.

Is the discrepancy getting at a long term impact? If so I don't see where that is hinted in the stimulus . I thought D was too ambiguous.

Thankyou
Sarah
Hello,

There is nothing about a long-term impact, especially.
B is bad enough that it is almost an Opposite answer. You want an answer that deals with both sides of the paradox, but answer B makes it plain that the local economy is hardly hurt by the mall at all. ("tourists who are drawn specifically by the mall and who would not have visited the community had that mall not been built.") So the rest of the local economy isn't really hurt, then why doesn't the economic boost from the mall help the community as much as it should?

David
 akanshalsat
  • Posts: 104
  • Joined: Dec 20, 2017
|
#59095
Hello! Still very very confused as to why B doesnt work?
 Malila Robinson
PowerScore Staff
  • PowerScore Staff
  • Posts: 296
  • Joined: Feb 01, 2018
|
#62990
Hi akanshalsat,
Can you describe where your confusion is coming from in a bit more detail? We are happy to try to explain it in a different way than what has already been done if you can give us a little more information on what part of our previous responses did not work for you.
-Malila

Get the most out of your LSAT Prep Plus subscription.

Analyze and track your performance with our Testing and Analytics Package.